Linear Transformations of Matrices

Click For Summary
The discussion focuses on solving for the linear transformations T(e1) and T(e2) of matrices. The user is attempting to find the coefficients alpha and beta for both e1 and e2 to construct the standard matrix. After some calculations, they determine T(e1) as <0, 1, 2> and T(e2) as <3/2, 1/2, -1/2>. The final standard matrix is confirmed as correct, and it is suggested to verify the results by multiplying the matrix with the vectors <1, 2> and <3, 4>. Overall, the process of deriving the standard matrix from the transformations is clarified.
schmidtc89
Messages
5
Reaction score
0

Homework Statement



attachment.php?attachmentid=29198&d=1287526189.jpg


The Attempt at a Solution



Code:
I think I first need to find  T(e[SUB]2[/SUB])=? and T(e[SUB]2[/SUB])=? and then combine those into a matrix. 

I am having trouble starting to solve for T(e[SUB]1[/SUB]) and T(e[SUB]2[/SUB])

so far I have   [1] = alpha [1]  + beta [3]
                [0]         [2]         [4]
                                     

I am trying to solve for alpha and beta to find e1


for e2 I have

so far I have   [0] = alpha [1]  + beta [3]
                [1]         [2]         [4]
                           
I am trying to solve for alpha and beta to find e2.

Once I solve these for T(e[SUB]1[/SUB]) & T(e[SUB]2[/SUB]) do I just combine the vectors for the standard matrix?
 
Guidance would be great.

Thank You.
 

Attachments

  • Untitled.jpg
    Untitled.jpg
    14.4 KB · Views: 538
Last edited:
Physics news on Phys.org
T(<1, 2> = T(1<1, 0>) + 2<0, 1>) and T(<3, 4> = T(3<1, 0>) + 4<0, 1>), right?
 
Mark44 said:
T(<1, 2> = T(1<1, 0>) + 2<0, 1>) and T(<3, 4> = T(3<1, 0>) + 4<0, 1>), right?

Yes the above is right and makes sense.
 
I am not sure if I did it right.

For t(e1) I got <0,1,2>
For t(e2) I got <3/2,1/2,-1/2>

Standard matrix T( e1 e2 ) -----> <0,1,2><3/2,1/2,-1/2>
 
Yes, this is correct. You can check by multiplying your 3 x 2 matrix with the two vectors <1, 2> and <3, 4> (transposed).
 
Thanks
 
Question: A clock's minute hand has length 4 and its hour hand has length 3. What is the distance between the tips at the moment when it is increasing most rapidly?(Putnam Exam Question) Answer: Making assumption that both the hands moves at constant angular velocities, the answer is ## \sqrt{7} .## But don't you think this assumption is somewhat doubtful and wrong?

Similar threads

Replies
4
Views
2K
Replies
8
Views
2K
  • · Replies 7 ·
Replies
7
Views
2K
  • · Replies 7 ·
Replies
7
Views
5K
Replies
1
Views
1K
  • · Replies 9 ·
Replies
9
Views
2K
  • · Replies 3 ·
Replies
3
Views
2K
  • · Replies 9 ·
Replies
9
Views
3K
  • · Replies 10 ·
Replies
10
Views
2K
Replies
5
Views
2K